LSAT and Law School Admissions Forum

Get expert LSAT preparation and law school admissions advice from PowerScore Test Preparation.

 DJYoungCorduroy
  • Posts: 7
  • Joined: Oct 05, 2017
|
#40338
Amazing forum here!

When I diagram answer choice B, I chose the flower bee balm as the necessary condition and the abundant crop as the sufficient condition because of the word "enable."

The stimulus I have diagrammed correctly as:

Well-known actors :arrow: Does well at box office
WKA :arrow: WBO

And the error they commit is a mistaken negation.

But in answer choice B, because I diagrammed it as

Abundant Crop :arrow: Bee Balm
AC :arrow: BB

I took the conclusion to be the contrapositive, and thereby it did not seem flawed.

I guess I just got tripped up on the word "enable," so I wanted to ask if you could help me find another way of saying what is presented in answer B so I would get the sufficient and necessary condition right?

Thanks!
 Adam Tyson
PowerScore Staff
  • PowerScore Staff
  • Posts: 5153
  • Joined: Apr 14, 2011
|
#40343
Thanks for the question, and for the great user name, DJYoungCorduroy! First things first here, and that is that this question does NOT have conditional reasoning in it and there is no sufficient or necessary condition in the stimulus. It's close, but not quite conditional because of the use of "many", bringing us instead into the world of formal logic. Having well known actors is not sufficient for a movie to do well at the box office, because if it was then it wouldn't be many, but ALL. Conditional reasoning is absolute in that regard, whereas formal logic is just about "some" or "many" or "most".

That said, we diagram formal logic in much the same way as conditional reasoning, and if you have our full length course books you can find a module in the Lesson 8 homework about it as well as in the Online Student Center under that same lesson. There's also a chapter in the Logical Reasoning Bible about it. Those will help with the diagrams.

Now, as to the "enables" in answer B, that word doesn't really trigger anything conditional or formal. It means "makes possible", but doesn't tell us that it ever actually happens. What we do have in answer B is "often", which tracks with the "many" in the stimulus. "Many movies starring top actors" is equivalent to "often bee balm", while "will do well at the box office" is the same as "attracts bumblebees" (which, in turn, enable abundant crops). It's not a perfect match - I would have preferred something like "gardens that lack bee balm will not attract bumblebees" - but it's the best match of the bunch, because it does the formal logic equivalent to a mistaken negation, as you correctly analyzed.

Looked at using a test of abstraction, the argument says some A have characteristic B, and therefore a non-A will also be a non-B. This is bad logic, because "some" does not mean "only these". Answer B has the same sort of flaw, equating "some" with "only these".

Do some searches into formal logic and you will find it discussed in several more threads in this forum, as well as in your books. It's not tested as much as conditional reasoning, and it is a close cousin to it, but there are some key differences that are worth looking over. For example, what might look like a mistaken reversal would be totally valid in formal logic, like so:

Some dogs are pitbulls; therefore, some pitbulls are dogs.

Explore that and have fun. Good luck!
 icandy
  • Posts: 1
  • Joined: Jun 05, 2019
|
#65414
Hi,

I have a particular problems with these type of questions. I can understand what the flaw is, but at the end there is always two remaining similar answers with similar structures and I have a hard time discerning these. Is there any general rule that can be applied to these types?

Best,
 Brook Miscoski
PowerScore Staff
  • PowerScore Staff
  • Posts: 418
  • Joined: Sep 13, 2018
|
#65422
icandy,

I think your issue is that identifying that the stimulus contains formal logic is not the same as understanding the flaw in the stimulus. If you understand the flaw in the stimulus, you should not have trouble identifying the correct answer choice, as there is only one choice that contains a logical structure similar to the stimulus.

The way that you can improve at identifying the flaw in the stimulus and in the correct choice is to study the common LSAT flaws and study how to diagram formal logic statements, like you see in the discussion above. If you can correctly diagram the stimulus and each answer choice, only one choice will be similar. Diagramming can help you get beyond confusion about what the language means. You may reach a point where you only diagram the most complex formal logic, or you may need to always diagram formal logic to correctly follow it.

Our Logic Reasoning Bible and our online and classroom classes provide methodology.
User avatar
 Chantal
  • Posts: 7
  • Joined: Jun 22, 2021
|
#89871
Hello! I chose the right answer (B), but struggled to explain why (D) is wrong when reviewing. Would someone be able to break down the structure of answer choice (D) and explain why it's not right? Thank you in advance!
User avatar
 Beatrice Brown
PowerScore Staff
  • PowerScore Staff
  • Posts: 75
  • Joined: Jun 30, 2021
|
#89937
Hi Chantal! Happy to help you out with this :)

First, let's breakdown what the flaw in the stimulus is. Notably, the stimulus utilizes formal logic (which, as Adam pointed out above, is not the same as conditional logic, but there are some important similarities!). The stimulus argues that movies with unknown actors will likely not do well at the box office. What evidence is given for that claim? The evidence is that many movies that have top actors do well at the box office since the actors are already popular with a loyal following. The issue here is that just because something is true of most movies with top actors doesn't mean that the opposite is true of movies without known actors.

We can diagram this out with a "most" arrow:
Movies starring top actors :most: do well at box office
Movies starring unknown actors :most: do not do well at box office
Once we diagram it this way, we can see that this is akin to a mistaken negation in conditional logic. The author concludes that because most movies with a certain condition do well at the box office, then if a movie does not have that condition, it will be unlikely to do well at the box office. However, we can't conclude anything about movies without top actors from the evidence we are given.

We want to ensure that the correct answer choice has this same flaw, which answer choice (B) does and answer choice (D) does not.

Let's first look at why answer choice (D) is incorrect. The first thing we can look for in a parallel flaw question is whether the conclusions are paralleled. In answer choice (D), the conclusion is the last sentence. However, the conclusion of answer choice (D) does not truly parallel that of the stimulus. For example, the quantifier is different: answer choice (D) uses the word "might," and the stimulus uses the word "unlikely."

Moreover, we also want to make sure that the premises are parallel between the answer choice and the stimulus. Here, the premises are not parallel. In answer choice (D), the evidence that is given is that a certain tool "can" be useful in teaching math skills effectively. This is different than a "most" statement, which is what the stimulus uses.

Finally, the flaw in the stimulus isn't quite matched by answer choice (D). In answer choice (D), the argument concludes that teaching math may be more difficult without visual aids. What evidence is given for this conclusion? That visual aids can be useful to effectively teach math. The formal logic in the stimulus, then, isn't paralleled by the answer choice, and we don't have this same type of error akin to a mistaken negation present.

To see this a bit better, let's take a look at answer choice (B), which is the correct answer. First, the conclusion of answer choice (B) is the last sentence, which is parallel to the conclusion in the stimulus: the answer choice uses the phrase "usually do not," which is akin to the phrase "unlikely" in the stimulus. Second, the premises in the answer choice and the stimulus are parallel. Answer choice (B) tells us about something that is often true, and the stimulus tells us about something that is true of most movies with a certain characteristic; "often" and "most" can be considered parallel.

Finally, we can diagram answer choice (B) with formal logic to see how the flaw is the same:
Presence flower bee balm :most: attract bumble bees and abundant crops produced
No flower bee balm :most: abundant crops not produced
When we diagram out answer choice (B) using formal logic, we can see that it makes this same type of error that is like a mistaken negation!

To sum up, on a parallel flaw question, you want to ensure that the flaw is paralleled between the stimulus and the answer choice you select. However, it is also helpful to use techniques to eliminate answer choices that you would use on regular parallel reasoning questions, such as checking the conclusions and premises to make sure these are parallel. In answer choice (D), the premise and conclusion were not parallel to those in the stimulus, whereas those in answer choice (B) were.

I hope this helps, and let me know if you have any other questions!

Get the most out of your LSAT Prep Plus subscription.

Analyze and track your performance with our Testing and Analytics Package.